Menu Close

2-289-1-2-17-1-2-a-1-2-a-2-2-a-3-2-a-k-Find-the-value-of-k-




Question Number 134689 by benjo_mathlover last updated on 06/Mar/21
 ((2^(289) +1)/(2^(17) +1)) = 2^a_1   + 2^a_2   + 2^a_3   + ... + 2^a_k    Find the value of k.
$$\:\frac{\mathrm{2}^{\mathrm{289}} +\mathrm{1}}{\mathrm{2}^{\mathrm{17}} +\mathrm{1}}\:=\:\mathrm{2}^{{a}_{\mathrm{1}} } \:+\:\mathrm{2}^{{a}_{\mathrm{2}} } \:+\:\mathrm{2}^{{a}_{\mathrm{3}} } \:+\:…\:+\:\mathrm{2}^{{a}_{{k}} } \\ $$$$\mathrm{Find}\:\mathrm{the}\:\mathrm{value}\:\mathrm{of}\:{k}. \\ $$
Answered by EDWIN88 last updated on 06/Mar/21
Let 2^(17)  = y  ⇒ ((y^(17) +1)/(y+1)) = y^(16) −y^(15) +y^(14) −y^(13) +y^(12) −y^(11) +...+1   = y^(15) (y−1)+y^(13) (y−1)+y^(11) (y−1)+...+y(y−1) +1   = (y^(15) +y^(13) +y^(11) +y^9 +...+y)(y−1)+1   = (2^(255) +2^(221) +2^(187) +...+2^(17) )(2^(17) −1)+1   = (2^(255) +2^(221) +2^(187) +...+2^(17) )_(8) (2^(16) +2^(15) +2^(14) +...+2+1)_(17) +1  so we get k = 8×17+1 = 137
$$\mathrm{Let}\:\mathrm{2}^{\mathrm{17}} \:=\:\mathrm{y} \\ $$$$\Rightarrow\:\frac{\mathrm{y}^{\mathrm{17}} +\mathrm{1}}{\mathrm{y}+\mathrm{1}}\:=\:\mathrm{y}^{\mathrm{16}} −\mathrm{y}^{\mathrm{15}} +\mathrm{y}^{\mathrm{14}} −\mathrm{y}^{\mathrm{13}} +\mathrm{y}^{\mathrm{12}} −\mathrm{y}^{\mathrm{11}} +…+\mathrm{1} \\ $$$$\:=\:\mathrm{y}^{\mathrm{15}} \left(\mathrm{y}−\mathrm{1}\right)+\mathrm{y}^{\mathrm{13}} \left(\mathrm{y}−\mathrm{1}\right)+\mathrm{y}^{\mathrm{11}} \left(\mathrm{y}−\mathrm{1}\right)+…+\mathrm{y}\left(\mathrm{y}−\mathrm{1}\right)\:+\mathrm{1} \\ $$$$\:=\:\left(\mathrm{y}^{\mathrm{15}} +\mathrm{y}^{\mathrm{13}} +\mathrm{y}^{\mathrm{11}} +\mathrm{y}^{\mathrm{9}} +…+\mathrm{y}\right)\left(\mathrm{y}−\mathrm{1}\right)+\mathrm{1} \\ $$$$\:=\:\left(\mathrm{2}^{\mathrm{255}} +\mathrm{2}^{\mathrm{221}} +\mathrm{2}^{\mathrm{187}} +…+\mathrm{2}^{\mathrm{17}} \right)\left(\mathrm{2}^{\mathrm{17}} −\mathrm{1}\right)+\mathrm{1} \\ $$$$\:=\:\underset{\mathrm{8}} {\underbrace{\left(\mathrm{2}^{\mathrm{255}} +\mathrm{2}^{\mathrm{221}} +\mathrm{2}^{\mathrm{187}} +…+\mathrm{2}^{\mathrm{17}} \right)}\underset{\mathrm{17}} {\left(\mathrm{2}^{\mathrm{16}} +\mathrm{2}^{\mathrm{15}} +\mathrm{2}^{\mathrm{14}} +…+\mathrm{2}+\mathrm{1}\right)}}+\mathrm{1} \\ $$$$\mathrm{so}\:\mathrm{we}\:\mathrm{get}\:{k}\:=\:\mathrm{8}×\mathrm{17}+\mathrm{1}\:=\:\mathrm{137} \\ $$

Leave a Reply

Your email address will not be published. Required fields are marked *